1

Imagine two positive charges in a space ship moving with a velocity,v with respect to an observer on earth.

according to the person in the spaceship,the electrostatic force between the charges is $F'=(\frac{1}{4π\epsilon_0})\times \frac{q_1q_2}{r^2} $

But if you are the observer on earth,then the equation becomes $F=\gamma \times F' =\gamma\times(\frac{1}{4π\epsilon_0})\times \frac{q_1q_2}{r^2} $ enter image description here
So,the force is greater in the frame on earth. But actually shouldn't the force be lesser because with respect to earth the charges are moving and hence have an inward magnetic attractive force which partially cancels the outwards repulsive force?

Sophile
  • 410

3 Answers3

1

But if you are the observer on earth,then the equation becomes $F=\gamma \times F' $

The above is not correct. The actual formula in this example would be $F= F' / \gamma$

In the frame of the person in the spaceship the force will indeed be as you have written $F' =(\frac{1}{4π\epsilon_0})\times \frac{q_1q_2}{r^2}$

EDIT : I have looked back on this and the rest of the answer is not accurate and just plain wrong. Hence , i am deleting the rest of what i had written.

The following question has a good answer which seems to be correct and provides a good answer for this
Is Magnetic force really the relativistic correction for electrostatic force?

silverrahul
  • 4,446
  • This does not answer the question. It only restates my earlier comment without reference. – my2cts Apr 25 '21 at 15:21
  • Fair enough . i had read your comment to that answer, not your comment to the original question.. I will edit my answer to make it complete . – silverrahul Apr 25 '21 at 15:23
  • But $F=\gamma ma $ Or $\frac{F}{a} = \gamma m $ since $\gamma$ is greater for the observer on earth,$\frac{F}{a}$ should also be greater.and as acceleration 'a' is same for both observers,this must mean that Force in his frame must be larger. How is this possible if $F=\frac{ F'}{\gamma}$ ? – Sophile Apr 26 '21 at 07:01
  • @GeneralPhysics " acceleration 'a' is same for both observers " This is not true. Acceleration is not a lorentz invariant . Acceleration of each charge would be higher in the spaceship frame, than in the earth frame – silverrahul Apr 26 '21 at 07:10
  • oh...I thought acceleration was lorentz invariant..Thanks for the clarification – Sophile Apr 26 '21 at 07:31
  • @GeneralPhysics Was my answer helpful ? Otherwise , i will be deleting it. I am not satisfied with my answer – silverrahul Apr 26 '21 at 07:51
  • Your answer was very helpfull.Don't delete it.But i have one doubt.When i simplified the last equation,I got $\frac{F'}{\gamma^2} $ instead of F′/γ. Where did I go wrong? – Sophile Apr 26 '21 at 09:47
  • Yes, EXACTLY. This is why i wanted to delete it. I also tried that and realised that , I must be missing something somewhere. I have been looking into this and asking about this. I will edit my answer, when i figure it out. My initial feeling is the formula for magnetic force between 2 moving point charges, might be wrong. I am not sure that formula is correct. I found that formula somewhere, but i cannot find that formula anywhere else. So , that formula might be wrong. What do you think about that specific part ? Do you think formula for magnetic force i have used is correct ? – silverrahul Apr 26 '21 at 09:51
  • Ok. Where did you found the derivation from? Did you check the source again? I had found a derivation of biot savart's law from lorentz transform of force on youtube.This is the video https://youtu.be/a_OOrELS_70 In this video he takes $\gamma=1$. Thats how he did that.. – Sophile Apr 26 '21 at 10:19
  • @silverrahul No,I dont think the equation for magnetic force is wrong. – Sophile Apr 26 '21 at 10:34
  • @GeneralPhysics I think i found what i was missing. When i understand it properly, I will edit my answer – silverrahul Apr 26 '21 at 14:18
  • @silverrahul what is it that you think is missing?Can you share it with me so that i could also work on it? Im just curious:) – Sophile Apr 27 '21 at 07:47
  • @GeneralPhysics Well , for one thing, the electric force between the 2 charges would also change in different frames. I could not figure out , how the whole thing comes together. But , the electric force between the 2 charges also changes , as observed in the earth frame. It wont stay the same as it was in the spaceship frame. Look it up. I could not understand everything completely – silverrahul Apr 27 '21 at 08:22
  • @GeneralPhysics I was able to find this exact problem in Purcell E.M., Morin D.J. - Electricity and Magnetism-Cambridge University Press (2013) . But i could not understand completely. Try downloading it from here, and see if you can understand shit http://library.lol/main/EC286BB084748909C5324C5A5FCA3A05 – silverrahul Apr 27 '21 at 08:25
  • @silverrahul I read that.But i only found the section where they talk about charge surfaces or array or charges and not two point charges..Anyway i asked this as a seperate question here https://physics.stackexchange.com/q/632668/226419 – Sophile Apr 28 '21 at 16:23
  • @silverrahul Your answer was not plain wrong.It only missed a gamma.You lead me to the correct answer.Thanks for everything... – Sophile Apr 29 '21 at 10:51
  • @GeneralPhysics Yes. But it was starting to get downvotes and i was getting -ve points. That is why i removed that part. I think i will just out a link to the other question. – silverrahul Apr 29 '21 at 10:52
  • @GeneralPhysics Some people are just petty. If they feel slighted by something you say, they just go and downvote your answers. – silverrahul Apr 29 '21 at 11:12
1

A common class of error, in these sorts of problems, is getting the transformation backwards and putting $\gamma$ where you should have $1/\gamma$. That's what's happened to you here.

To see this intuitively, consider a pair of opposite charges instead of same-sign charges. Released from rest, the opposite charges will accelerate towards each other and collide after some finite time. An observer in a different frame will see this collision happen more slowly, due to time dilation.

How the moving observer explains this delayed collision is sensitive to the geometry of the problem. If the line between the charges is perpendicular to the velocity, as in your diagram, the moving observer believes there is a magnetic interaction between the charges which counteracts their electrical attraction. (A classic problem from Griffiths: consider parallel line charges, and find the speed at which the magnetic repulsion exactly cancels the electrical attraction.) If the line between your point charges were parallel to the velocity vector, there would be zero magnetic force between them, but the electrical attraction would have been reduced thanks to the Lorentz boost. At other angles, the boosted electrical force and the magnetic interaction conspire together so that your opposite charges collide after exactly the right dilated amount of time.

rob
  • 89,569
0

Since the vector F is perpendicular to v it is not affected by the Lorentz transformation and it is the same in both reference frames.

You can yourself a big favour by using the [covariant formulation of electromagnetism][1]. The force is given by $$f^\mu = F^{\mu\nu} j_\nu$$ where $F^{\mu\nu}$ is the Lorentz covariant, antisymmetric force tensor and $j^\nu$ the current. Form this it is clear that you don't have to worry about the detailed transformation of the fields. You only have to transform the force. The Lorentz transformation is $$ t' = \gamma \left( t - \frac{vx}{c^2} \right) \\ x' = \gamma \left( x - v t \right)\\ y' = y \\ z' = z \,. $$ This shows that if v is along x, then the perpendicular components of the force, unlike those of the antisymmetric tensor fields, are not affected. [1]: https://en.wikipedia.org/wiki/Covariant_formulation_of_classical_electromagnetism

my2cts
  • 24,097
  • Oh..Thanks for the reply – Sophile Apr 25 '21 at 13:38
  • That does not really matter. He could just as easily draw the 2 charges placed along the axis of the spaceship, in such a way, that the line joining them is parallel to v – silverrahul Apr 25 '21 at 13:54
  • But then,you need to take into account lorentz contraction.. – Sophile Apr 25 '21 at 14:22
  • 1
    @silverrahul That would be an entirely different question with a different answer. In my opinion it does matter what the actual question is. – my2cts Apr 25 '21 at 15:14
  • @my2cts Okay, if you are talking about the specific homework question, then of course it matters. But , if you are talking about the general concept of how to think about the forces between them, then he could just as easily ask, if the 2 charges are placed along the axis of the ship , such that the line joining them is parallel to v, then the lorentz contraction means they are closer, hence electrostatic force increases . And you have got the same question again ? This is what i meant , when i said, that F being perpendiclar to V is not the whole answer – silverrahul Apr 25 '21 at 15:19
  • 1
    @silverrahul I am just sticking to the talking points raised by the OP. – my2cts Apr 25 '21 at 15:22
  • @silverrahul This is not a homework question. If the charges are aligned horizontally,then there would be no magnetic forces.That is why i had no more doubts .Anyway thanks for the consideration.Actually you have a point .He did answer my specific question,but i have still doubts remaining of how the lorentz contraction affects the forces between charges.Anyway i will ask it as a different question.I will leave the link here – Sophile Apr 26 '21 at 05:16
  • Okay, maybe homework question means something specific in the context of this site. My point was more along the lines of , homework question = THIS specific question with this specific setup as opposed to the general concept behind what would happen with moving charges. WHat specifically is your doubt about how Lorentz contraction affects forces ? You can ask here, i think – silverrahul Apr 26 '21 at 05:28
  • @my2cts But we still have to transform the forces,right? – Sophile Apr 26 '21 at 10:38
  • @ProfRob I added the details of the Lorentz transformation for completeness. Perpendicular components of the force are unaffected. – my2cts Apr 29 '21 at 15:15
  • I'm afraid not. Showing the Lorentz transformations of displacement isn't relevant. The transformation of force is $F' = F_{\parallel} + \gamma^{-1} F_{\perp}$. – ProfRob Apr 29 '21 at 15:58
  • @ProfRob I don't know how you define $F$, but the force as defined by $$f^\mu = F^{\mu\nu} j_\nu$$ is a Lorentz vector and transforms as such. – my2cts Apr 29 '21 at 16:41
  • Why are you talking about 4 vectors? This question is asking about forces (3-vectors). https://web.archive.org/web/20090226225531/http://www.hep.princeton.edu/~mcdonald/examples/EM/ganley_ajp_31_510_62.pdf http://www.sciencebits.com/Transformation-Forces-Relativity – ProfRob Apr 29 '21 at 16:44
  • And here is a demonstration using the transformed electromagnetic fields. https://physics.stackexchange.com/questions/307685/relativistic-electromagnetism-and-electromagnetic-forces-on-2-protons and others https://physics.stackexchange.com/a/71391/43351 https://physics.stackexchange.com/questions/248058/confusion-on-relativistic-electromagnetism-of-two-charge-particles?noredirect=1&lq=1 – ProfRob Apr 29 '21 at 16:48